simplify (2^2×4^-2)×5^8+2+3^0​

Answers

Answer 1
Here you go
I am sorry if it’s wrong
Answer 2


This is my best
Sorry if missed it.

Related Questions

What is the value of x in the triangle below?

A. 18

B. 15

C. 6

D. 45

Answers

Answer:

it is b I already took it I just checked again

PLEASE HELP WILL GIVE BRAINSLIST

Answers

Answer: 3. 55° 2. 225° 1. 125°

Step-by-step explanation: don't want brainliest

write a inequality phrase: you have to be at least 21 years of age

Answers

Answer:

[tex]a\geq 21[/tex]

Step-by-step explanation:

"At least" is inclusive, so you can be 21 years old.

Answer:

a≥ 21

Step-by-step explanation:

Let a = age

At least means greater than or equal to

a≥ 21

Select all that apply.
The slope of the graph of a direct variation function is -3. Which of the following is true?

The equation is y = -3x.
The function is linear.
The equation of the function is y = -x .
The point (2, -6) is on the graph of the function.

Answers

a and b are correct

c is wrong

d can't be decided, because it can be true for one case, but not for others.

so only go for a and b

Answer:

B. The function is linear. (A and D COULD work)

Step-by-step explanation:

If the graph of a direct variation function has a slope of -3, then we know that the function is linear, and that m in the equation y = mx + b is -3. However, option A could be incorrect since we don't know if there is a y-intercept or not. C is wrong because of -3 being the slope, and D would only be true if A was also correct. So that leaves B. (If you know that there is more than one option, then A, B, and D are correct.)

A library has 160 books. 3/8 are story books, 1/4 are poetry books and the remainder are books. What fraction is comic books?

Answers

Answer:

1/8

Step-by-step explanation:

1/4*2/2=4/8

4/8+3/8=7/8

1-7/8=1/8

solve
[tex]100 {5}^{2} - 500 {5}^{6} [/tex]
Please help before 9:00 pm​

Answers

1005 to the power of 2 is 1010025 sorry if i'm wrong, and 5005 to the power of 6 is 1.5718985e+22. so 1.5718985e+22-1010025=-1.5718985e+22. wow my answer is so random ofc it's not correct lollllll i'm sorry i'm horrible at math and i barely understood this equation so i am super super super sry :(

On a piece of paper, graph y< - 1/2 x + 1

Answers

Answer:

Image below

Step-by-step explanation:

Make sure you draw it with a dotted line.

can someone help me pls

Answers

Answer: No solution

Step-by-step explanation:

First, rearrange the second function:

[tex]3x - y = 2 \\-y=-3x+2\\y=3x-2[/tex]

Because the two equations share the same slope of 3, they're parallel lines, that means they don't ever intercept. Therefore, there are no solutions.

What is the equation of the following line? Be sure to scroll down first to see
all answer options.
10
(-4,8)
(0,0)
. 10
10

Answers

The answer is y= -2x.

The equation of the line passing through points (0, 0) and (-4, 8) is y = -2x

What is an equation of line?

The equation of line is an algebraic form of representing the set of points, which together form a line in a coordinate system.

Given is a graph of a line passing through points (0, 0) and (-4, 8), we need to find the equation of the line,

We know that, the equation of a line passing through two points is given by,

y-y₁ = y₂-y₁ / x₂-x₁ (x-x₁)

Here, (x₁, y₁) and (x₂, y₂) are (0, 0) and (-4, 8)

So, the equation of the line =

y-0 = 8/(-4) (x-0)

y = -2x

Hence, the equation of the line passing through points (0, 0) and (-4, 8) is y = -2x

Learn more about equation of line, click;

https://brainly.com/question/21511618

#SPJ7

Urgent i need help!!…….

Answers

Answer:

Step-by-step explanation:

These are similar triangles. We know that because we know that all right triangles are similar. The height of the red one is 8 and the height of the blue one is 4; that means that the red one is twice the size of the blue one; likewise, the blue one is half the size of the red one. That means that ALL the measurements of these triangles exist in that ratio...even the base of the blue one. If the base of the red one is 3, and the red one is twice the size of the blue one, then the base of the blue one is 3/2 or 1.5. I can't see your choices because they are too small.

it should be 1.5……………. if it’s wrong sorry

what is the quotient

Answers

Answer:

The quotient is 4x+5  and  the remainder is -13

Step-by-step explanation:

Using synthetic division

-5        4      25   12

Bringing down the 4 and multiplying

-5        4      25   12

        ↓       -20 -25

    ----------------------

         4        5    -13

The quotient is 4x+5  and  the remainder is -13

Answer:

  4x   +5    [tex]\frac{-13}{x+5}[/tex]

Step-by-step explanation:

          .           4x   +5    [tex]\frac{-13}{x+5}[/tex]

x + 5 | 4x^2 + 25x + 12

        - 4x^2 + 20x

                         5x + 12

                         5x + 25

                               -13

Keith used the following steps to find the inverse of f, but he thinks he made a error
f(x) = 7x + 5

Answers

Answer:

Step-by-step explanation:

[tex]\Large \boldsymbol{} f(x) \ \ inverse \ \ function \ \ (f(x))^{-1} \\\\ y=7x+5 \\\\x=7y+5 \\\\ y=\dfrac{x-5}{7} \ \ or \ \ f(x)^{-1}= \dfrac{x-5}{7}[/tex]

Which number produces a rational number when added to 0.25? ​

Answers

The step of adding it to 0.25 (or 25/100, or 1/4) is indeed not relevant. 0.25 is rational.

A rational number added to a rational number gives a rational number. A rational added to an irrational number doesn't.

so effectively the task can be reduced to "wich one of these is a rational number?"

And that's only true for A. 2/9

All others can't be represented as a discrete/finite fraction / ratio.

The sum of two rational numbers is a rational number thus option (A) is correct.

What is an irrational number?

Any real number that cannot be written as the quotient of two integers, p/q, where p and q are both integers, is referred to as an irrational number.

For example, √2, and √3 are irrational numbers because they cannot be written as p/q where p and q both should be integers.

The sum of two rational numbers gives the rational number.

Since 2/9 is written in p/q where 2 and 9 both are integers thus it will be rational.

So, 2/9 + 0.25 = 2/9 + 1/4

(8 + 9)/36 = 17/36 here 17 and 36 both are integers thus it will be rational.

Hence "A rational number is the result of two rational numbers".

For more about the irrational number,

https://brainly.com/question/4031928

#SPJ2

*Please Help!*

What is the volume of water, to the nearest tenth of a cubic metre, that would fill this spa tub?

First cylinder= 0.75m diameter, 0.80m height

Cylinder Underneath= 1.25m diameter, 0.70m height

Semi Sphere that holds both cylinders= 3m long

Answers

Answer:

The volume of water that will fill the spa tub is 5.9 cubic meters.

Step-by-step explanation:

Volume of water that would fill the spa tub = volume of semi sphere - (volume of the first cylinder + volume of the second cylinder)

i. volume of first cylinder = [tex]\pi[/tex][tex]r^{2}[/tex]h

where r is the radius and h is the height of the cylinder.

r = [tex]\frac{0.75}{2}[/tex] = [tex]\frac{3}{8}[/tex]

 = 0.375 m

h = 0.80 m

volume of the first cylinder = [tex]\frac{22}{7}[/tex] x [tex](\frac{3}{8} )^{2}[/tex] x 0.8

                                        = 0.3536 cubic meters

ii. volume of the cylinder underneath = [tex]\pi[/tex][tex]r^{2}[/tex]h

r = [tex]\frac{1.25}{2}[/tex] = [tex]\frac{5}{8}[/tex]

 = 0.625

h = 0.70 m

volume of the cylinder underneath = [tex]\frac{22}{7}[/tex] x [tex](\frac{5}{8}) ^{2}[/tex] x 0.7

                                                      = 0.8594 cubic meters

iii. volume of the semi sphere = [tex]\frac{2}{3}[/tex] [tex]\pi[/tex][tex]r^{3}[/tex]

where r is the radius = 1.5 m

volume of the semi sphere = [tex]\frac{2}{3}[/tex] x [tex]\frac{22}{7}[/tex] x [tex](1.5)^{3}[/tex]

                                             = 7.0714 cubic meters

Thus,

volume of the water to fill the spa tub = 7.0714 - (0.3536 + 0.8594)

                                                     = 5.8584

The volume of water that will fill the spa tub is 5.9 cubic meters.

Explain how to solve 5^(x-2)= 8 using the change of base formula

Answers

Answer:

x = 3.3

Step-by-step explanation:

A equation is given to us and we need to solve out for x. The given equation is ,

[tex]\sf\longrightarrow 5^{x -2}= 8 [/tex]

Take log on both sides with base as " 10" . We have ,

[tex]\sf\longrightarrow log_{10} 5^{x-2}= log_{10}\ 8[/tex]

Simplify using the property of log , [tex]\sf log a^m = m log a [/tex] , we have ,

[tex]\sf\longrightarrow ( x -2) log_{10} 5 = log_{10} 8 [/tex]

Simplify ,

[tex]\sf\longrightarrow ( x -2 ) log_{10}5 = log_{10} 2^3[/tex]

Again simplify using the property of log ,

[tex]\sf\longrightarrow (x-2) log 5 = 3 log 2[/tex]

We know that log 5 = 0.69 and log 2 = 0.301 , on substituting this , we have ,

[tex]\sf\longrightarrow ( x - 2 ) = \dfrac{ 3\times 0.301}{0.69}[/tex]

Simplify the RHS ,

[tex]\sf\longrightarrow x - 2 = 1.30 [/tex]

Add 2 both sides ,

[tex]\sf\longrightarrow \boxed{\blue{\sf x = 3.30}}[/tex]

Hence the Value of x is 3.30 .

Answer:

its actually 3.292 because we round to the nearest thousandth and thats not even the equation you use above

Step-by-step explanation:

For this equation we use the formula log a^m=m (log a) so the equation will be written as log 5 (5^x-2) = log 5 (8). You use the base, which is 5, and use log to base 5 on both sides of the equation. Then you take the exponent " x-2" and write( x-2) log 5 (5) = log 5(8). Since log a =1, you multiply that 1 by x-2, which keeps it x-2. Making the equation x-2 = log 5 (8). Next, we use the change of the base properties with the formula log b^y= log y/ log b. The equation will be written as x-2 = log 8/ log 5, since 5 is the base it stays in the bottom or basement. We then add +2 to both sides of x-2 and log 8/ log 5. To solve this equation, you can find out what log 8 and log 5 are and divide those and add +2 to solve. So log 8 = 0.903 and log 5 = 0.698970 and divide those to get 1.29190 +2 and you get the answer rounded as 3.292. 

v=u + 2at
Where v is the final velocity (in m/s), u is the initial velocity (in m/s), a is the
acceleration (in m/s?) and t is the time in seconds).
Find v when u is 35 m/s, a is 28 m/s2, and t is 58 seconds.

Answers

Answer:

3283m/s

Step-by-step explanation:

V=U+2at

V=35+2(28)(58)

V=35+3248

V=3283m/s

Gary's income and expenses in a month are 4,000 and 3,360 respectively.
a) What percentage of his income are expenses?
b) What percentage of his income is savings?
(SHOW YOUR WORK!)

Answers

Answer:

84%

Step-by-step explanation:

3360/4000=.84

.84=84%

a) Arrange the following numbers so that the sum of any two neighbouring numbers is a perfect square.
7 , 11 , 9 , 14 , 16 , 2 , 25
Ex. 3+6 = 3^2 and 6+10 = 4^2

b) Find a way to arrange all of the intervenes from 1 to 17 to a list with the property from part a)

Answers

I'll do part (a) to get you started.

One possible answer to part (a) is 25,11,14,2,7,9,16

There are likely other possible answers.

The explanation is below.

==============================================================

Let's say 7 is the anchor value and we want to see which values could be its next door neighbor.

It can't be 11 since 7+11 = 18 isn't a perfect square.But 9 works because 7+9 = 16 is a perfect square (4^2 = 16)14 doesn't work because 7+14 = 21 isn't a perfect square16 doesn't work either since 7+16 = 23 isn't a perfect square2 however does work since 7+2 = 9 is a perfect squareLastly, 25 won't work because 7+25 = 32 isn't a perfect square

To summarize this subsection, the anchor value 7 could have the neighbors 9 and 2.

So we could have 2,7,9 or 9,7,2 as a subsequence. We'll keep this in mind for later.

------------------------------------------------

Now we'll make 11 the anchor. We already checked 7 and it doesn't work.

9 doesn't work either because 11+9 = 20 isn't a perfect square

11+14 = 25 does work11+16 = 27 doesn't work11+2 = 13 doesn't work11+25 = 36 does work

Of that list, only 14 and 25 are possible neighbors of the anchor value 11.

So we could have the subsequence 14,11,25 or 25,11,14.

------------------------------------------------

If 9 is the anchor, then,

9+7 = 16 works like we found earlier (section 1)9+11 = 20 doesn't work9+14 = 23 doesn't work9+16 = 25 does work9+2 = 11 doesn't work9+25 = 34 doesn't work

The values 7 and 16 are possible neighbors of 9. We could have the subsequence 7,9,16 or 16,9,7

Let's go back to the subsequence 2,7,9 and tack 16 at the end to get 2,7,9,16

------------------------------------------------

If 14 is the anchor, then,

7 doesn't work because 14+7 = 2111 does work either because 14+11 = 259 doesn't work because 14+9 = 2316 doesn't work because 14+16 = 302 does work because 14+2 = 1625 doesn't work since 14+25 = 39

We could have the subsequence 11,14,2 or 2,14,11

Let's go with the first option and stick "11,14,2" in front of "2,7,9,16" to end up with the larger subsequence 11,14,2,7,9,16

We can then stick 25 at the front because 25+11 = 36 is a perfect square.

------------------------------------------------

So one possible sequence of values is 25,11,14,2,7,9,16

Here's the verification

25+11 = 36 is a perfect square (6^2 = 36)11+14 = 25 is a perfect square (5^2 = 25)14+2 = 16 is a perfect square (4^2 = 16)2+7 = 9 is a perfect square (3^2 = 9)7+9 = 16 is a perfect square (4^2 = 16)9+16 = 25 is a perfect square (5^2 = 25)

Each pair of adjacent terms add up to a perfect square, so the answer is confirmed.

There are probably other solutions as well.

Side note: The video math channel "Numberphile" has a video discussing this topic in which you might be interested in. Search out "The square sum problem" with quotes (the presenter/teacher in the video is Matt Parker).

Answer:

25, 11, 14, 2, 7, 9, 16.

Step-by-step explanation:

25 + 11 = 6^2

11 + 14 = 5^2

14 + 2 = 4^2

2 + 7 = 3^2

7 + 9 = 4^2

9 + 16 = 5^2

Which digit in 123.456 has highest value?
Digit
6

5

4
1

Answers

Which digit in 123.456 has highest value?

answer = 1....

Answer:

1

Step-by-step explanation:

The 1 is in the hundredth's place and it has the highest value

Please solve this problem 9p-10=6+p?

Answers

Answer:

p=2

Step-by-step explanation:

9p-10=6+p

Subtract p from each side

9p-p-10=6+p-p

8p -10 =6

Add 10 to each side

8p -10+10 =6+10

8p =16

Divide each side by 8

8p/8 = 16/8

p =2

Answer:

p = 2

Step-by-step explanation:

1. 9p - 10 = 6 + p

2. 8p - 10 = 6

3. 8p = 16

4. p = 2

Is this true or false

Answers

Answer:

I say it's TRUE

Step-by-step explanation:

I hope this help you

I assume that it’s true

A cash register at a store contains $277 bills. There are six more $5 bills than $10 bills. The number of $1bills is two more than 24 times the number of $10 bills. How many bills of each kind are there

Answers

Answer: See explanation

Step-by-step explanation:

Your question isn't well written, the store contains $227 and not $277.

let c be the number of $1 bills

let f be the number of $5 bills

let t be the number of $10 bills.

Therefore, the equation will be:

1c + 5f + 10t = 227

Since there are six more $5 bills than $10 bills, therefore,

f = t+6

Also, the number of $1 bills is two more than 24 times the number of $10 bills and this will be:

c = 24t + 2

Now, we substitute the equation for f and c into the main equation and this will be:

1c + 5f + 10t = 277

1(24t + 2) + 5(t + 6) + 10t = 227

24t + 2 + 5t + 30 + 10t = 227

39t + 32 = 227

39t = 227 - 32

39t = 195

t = 195/39

t = 5

Therefore, there are 5 $10 bills

Since c = 24t + 2

c = 24(5) + 2

c = 120 + 2

c = 122

Therefore, there are 122 $1 bills

Since f = t+6

f = 5+6

f = 11

Therefore, there are 11 $5 bills.

Can someone help me with this math homework please!

Answers

9514 1404 393

Answer:

7/3-3y = 7/3x - 3

Step-by-step explanation:

1) The slope is given by the formula ...

  m = (y2 -y1)/(x2 -x1)

  m = (4 -(-3))/(3 -0) = 7/3

__

2) The y-intercept is the point where x=0:

  (x, y) = (0, -3) . . . . . y = -3

__

3) Then the slope-intercept equation is ...

  y = mx + b . . . . . . . line with slope m and y-intercept b

  y = 7/3x -3

Find the center and radius of the circle whose equation is x 2 + y 2 - 6x - 2y + 4 = 0

Answers

Answer:

[tex]{ \tt{ {x}^{2} + {y}^{2} - 6x - 2y + 4 = 0 }} [/tex]

Center:

[tex]{ \tt{(3, \: 1)}}[/tex]

Radius:

[tex]{ \tt{radius = \sqrt{ {g}^{2} + {f}^{2} - c } }} \\ = \sqrt{36 + 4 - 4} \\ = 6 \: units[/tex]

expand this question (x+5)(x-3)

Answers

X•x + x•-3 + 5•x +5•-3
X^2+-3x+5x+-15
X^2+2x-15

a line has slope 3 and y-intercept 4

Answers

Answer:

y = 3x+4

Step-by-step explanation:

The slope intercept form of a line is

y = mx+b  where m is the slope and b is the y intercept

y = 3x+4

Help please…

If (tan^30-1/tan0-1)-sec^20+1=0, find cot0

Answers

Answer:

dl5fgtrrfhrruf ofender fkufgj jjgglr

Given m||n, find the value of x.
+
mm
(6x+4)
(4x-14)
Answer:
Submit Answer
attempt out of 2

Answers

Answer:

x = 19

Step-by-step explanation:

6x+4 + 4x - 14 = 180

10x    - 10 = 180

10 X = 190

x = 19

x=19 would be your final answer

what is 3^-8 x 3^-4 x 3^-2

Answers

Answer:

Value of given expression is 3⁻¹⁴

Step-by-step explanation:

Given expression:

3⁻⁸ x 3⁻⁴ x 3⁻²

Find:

Value of given expression

Computation:

⇒ 3⁻⁸ x 3⁻⁴ x 3⁻²

⇒ 3[⁻⁸⁻⁴⁻²]

⇒ 3⁻¹⁴

Value of given expression is 3⁻¹⁴

Suppose f(x)=x^3 what is the graph of f(x+3)

Answers

Answer:

[tex]f(x + 3) = {(x + 3)}^{3} [/tex]

Shifted left with x-intercept at (-3,0) and the y-intercept at (0,27)

Other Questions
Please write a letter to yourself commenting on your achievements and indicating your plans to continue developing. Be specific, fair, and plan well. Do not forget to answer the following questions in your paper: Who are you as a Leader? What are your strong points and what are your developmental goals? What will you be able to contribute to your future organization? How? I need help ASAP!!Please help me Consider a hypothetical metal that has a density of 10.6 g/cm3, an atomic weight of 176.8 g/mol, and an atomic radius of 0.130 nm. Compute the atomic packing factor if the unit cell has tetragonal symmetry, values for the a and c lattice parameters are 0.570 and 0.341, respectively. Hi I need help :( I cant figure these out Refer to this design web of a car manufacturers experiment. Which of the following statements are true? Check all that apply.A confounding variable is the use of cruise control.A confounding variable is the weather conditions.A confounding variable is the participants reporting on their fuel efficiency.There are no confounding variables in this experiment.A confounding variable is the speed at which the participants drive on the course. To meet projected annual sales, Bluegill Manufacturers, Inc. needs to produce 75,000 machines for the year. The estimated January 1 inventory is 7,000 units, and the desired December 31 inventory is 12,000 units. What are projected sales units for the year? fill in the blank 1 units Use the formula (a+b)2=a2+2ab+b2 to expand the (4x+3y)2 Megan accelerates her skateboard from 0 m/s to 8 m/s in 2 seconds. What is the magnitude of the acceleration of the skateboard? O 8 m/s^2 O 16 m/s^2 O 2 m/s^2 O 4 m/s^2 do you go...school...foot or...bike 2/3 of a number is 20 less than the original number.find the number Why do factory farms want to keep consumers from knowing about the conditions under which animals are raised and what strategies do they use to keep us in the dark? What is the minimum perimeter of a rectangle with an area of 625 mm^2 What is the result of rounding 8,888,888 to the nearest ten? 3)Write an inequality for the graph below. If necessary, use= for.Kinda stuck and running out of time How would "In a Station of the Metro" change if the bough was bright and lively rather than wet and black? Which of the following is a way to demonstrate active listening?A. Do not look the speaker in the eyesB. Interrupt the speaker with questionsC. Plan your response while the speaker is talkingD. Watch the speaker's body language Question 2: thanks!!! Choose the word/phrase that best fits this blank below:You ________ touch anything electrical with wet hands.Select one:a.don't have tob.mustn'tc.needn't why Ethyl amine is more basic than aniline? A company must repay the bank $10,000 cash in three years for a loan. The loan agreement specifies 8% interest compounded annually. The present value factor for three years at 8% is 0.7938. How much cash did the company receive from the bank on the day they borrowed this money?A. $12,400.B. $9,200.C. $7,938.D. $7,600.E. $10,000.